Which one of the following, if true, would cast doubt on the position concerning innovation in software development t...

Sydney on September 9, 2019

Answer explanation

Can you please explain why D is correct and the rest are wrong. Thanks!

Reply
Create a free account to read and take part in forum discussions.

Already have an account? log in

Irina on September 9, 2019

@sydneycrudo123,

Great question. This is a weaken question, meaning the correct answer choice undermines the conclusion, and the remaining answer choices have no impact or strengthen the conclusion. The argument tells us that software patents impede innovation in software development. Let's look at the answer choices:

(A) Most patents have a duration of 20 years or less.

Incorrect. This fact likely strengthens the argument, 20 years is a long time in terms of software development, and a 20 year patent is likely to impede innovation for a given software product.

(B) Software companies that do not patent products generally offer products that are more reliable.

Incorrect. This fact has no impact or strengthens the argument if we are to assume that reliability correlates with innovation.

(C) Some proprietary vendors oppose patents for self interested reasons.

Incorrect. This fact has no impact on the validity of the argument because it fails to tell us anything the impact of patents on innovation.

(D) Software innovation will be less profitable if software could not be patented.

Correct. This fact tells us that patents encourage rather than impede innovation because without patents software innovation is not as profitable and thus there is less incentive to innovate. Since this fact demonstrates that patents are beneficial for software innovation, it weakens the argument.

(E) The main beneficiaries of patents are large corporations rather than individual innovators.

Incorrect. This fact strengthens the argument, if patents benefit only corporations not innovators, patents could only impede or have no impact on innovation.

Let me know if this makes sense and if you have any further questions.